[PDF] [PDF] Math 431 - Real Analysis I

(c) Show that the sequence xn is bounded below by 1 and above by 2 (d) Use (e) Use (d) in a proof to show that Sn is Cauchy and thus converges In the first case, if k = 0, then we wish to prove that k · f(x) = 0, the zero function, has limit 0



Previous PDF Next PDF





[PDF] Prove that ) is Cauchy using directly the definition of Cauchy

Assume that (xn)n∈N is a bounded sequence in R and that there exists x ∈ R such that any convergent subsequence (xni )i∈N converges to x Then limn→∞ xn 



[PDF] 14 Cauchy Sequence in R

A sequence xn ∈ R is said to converge to a limit x if • ∀ϵ > 0, ∃N s t n > N ⇒ xn − x < ϵ A sequence xn ∈ R is called Cauchy sequence if • ∀ϵ, ∃N s t n > N m > N ⇒ xn − xm < ϵ Every convergent sequence is a Cauchy sequence Proof



[PDF] Question 1 - Properties of Cauchy sequences Question 2

Show that xn → x as well; i e to prove that a Cauchy sequence is convergent, we only need i e any bounded sequence has a convergent subsequence Show 



[PDF] 1 Cauchy sequences - ntc see result

A sequence {an}is called a Cauchy sequence if for any given ϵ > 0, there exists N ∈ N Proof Since {an}forms a Cauchy sequence, for ϵ = 1 there exists N ∈ N such that (i) lima1/n = 1, if a > 0 (ii) limnαxn = 0, if x < 1 and α ∈ IR Solution: 



ON U-CAUCHY SEQUENCES - Project Euclid

If a sequence of points in X is U-convergent in X then it fulfills U-Cauchy condition Proof Let U − lim n→∞ xn = x and ε > 0 Thus A(ε/2) = {n ∈ N : ρ(xn, x) ≥



[PDF] details - Lecture summary

Definition A sequence (an) is said to be a Cauchy sequence iff for any ϵ > 0 there exists N such prove (over the course of 2 + ϵ lectures) the following theorem: If a subsequence of a Cauchy sequence converges to x, then the sequence



[PDF] be a metric space Let - School of Mathematics and Statistics

Let X = (X, d) be a metric space Let (xn) and (yn) be two sequences in X such that (yn) is a Cauchy sequence and d(xn,yn) → 0 as n → ∞ Prove that (i) (xn) is a 



[PDF] ANALYSIS I 9 The Cauchy Criterion - People Mathematical Institute

Every complex Cauchy sequence is convergent Proof Put zn = x + iy Then xn is Cauchy: xx − xm ⩽ zn − zm (as  



[PDF] Math 431 - Real Analysis I

(c) Show that the sequence xn is bounded below by 1 and above by 2 (d) Use (e) Use (d) in a proof to show that Sn is Cauchy and thus converges In the first case, if k = 0, then we wish to prove that k · f(x) = 0, the zero function, has limit 0



[PDF] 8 Completeness

If (X, d) is a complete metric space and Y is a closed subspace of X, then (Y,d) is complete Proof Let (xn) be a Cauchy sequence of points in Y Then (xn) also 

[PDF] show that x is a discrete random variable

[PDF] show that x is a markov chain

[PDF] show that x is a random variable

[PDF] show that [0

[PDF] show the mechanism of acid hydrolysis of ester

[PDF] show time zone cisco

[PDF] show ∞ n 2 1 n log np converges if and only if p > 1

[PDF] shredded workout plan pdf

[PDF] shredding diet

[PDF] shredding workout plan

[PDF] shrm furlough

[PDF] shuttle paris

[PDF] si clauses french examples

[PDF] si clauses french exercises

[PDF] si clauses french practice

Math 431 - Real Analysis I

Solutions to Homework due October 22

Question 1.Sequences are frequently givenrecursively, where a beginning termx1is specied and subse- quent terms can be found using a recursive relation. One such example is the sequence dened byx1= 1 and x n+1=p2 +xn: (a) Forn= 1;2;:::;10, computexn. A calculator may be helpful. (b) Show thatxnis a monotone increasing sequence. A proof by induction might be easiest. (c) Show that the sequencexnis bounded below by 1 and above by 2. (d) Use (b) and (c) to conclude thatxnconverges.

Solution 1.

(a)nx n11

21:4142131:8477641:9615751:9903661:9975971:9993981:9998591:99996101:99999(b) LetA(n) be the statement thatxn+1xn. We will show thatA(n) is true for alln1. First, note

thatx1= 1Thus, 2 +xk+12 +xk+2. Thus, x k+2=p2 +xk+1p2 +xk=xk+1: Thus,A(k+ 1) is true. So, by induction,A(n) is true for alln. (c) First, note that sincex1= 1 and thatxnis monotone increasing, thenxn1 for alln. For the other bound, we will use induction on the statementA(n) given byxn2 forn1. For the base case, notice thatx1= 1<2. Thus,A(1) holds. Now, assume thatA(k) holds. WE will show thatA(k+ 1) is true.

SinceA(k) is true, thenxk2. Thus,

x k+1=p2 +xkp2 + 2 = 2: Thus,A(k+ 1) is true. So, by induction,xkis bounded above by 2.

(d) By (b) and (c),xnis a bounded, monotone sequence; thus, by a theorem in class, it converges.Question 2.One very important class of sequences areseries, in which we add up the terms of a given

sequence. One such example is the following sequence: S n=nX k=01k!=10! +11! +12! +13! ++1n!: 1 (a) Forn= 0;1;2;:::;6, computeSn. Again, a calculator may be helpful; be sure to use several digits. (b) Show thatSnis monotone increasing. (c) Use induction to show that for alln1,n!2n1. (d) Use (c) to show that S n1 +nX k=112 k1: (e) Use well-known facts from Calculus II and the geometric series to show that 1 + nX k=112 k1<3: (f) Use (b), (d), and (e) to conclude thatSnconverges.

Solution 2.

(a)nS n01 12

22:532:6666642:70833352:716666662:7180555(b) Since 1=n!>0, then

S n+1=Sn+1(n+ 1)!Sn:

Thus,Snis monotone increasing.

(c) LetA(n) be the statement thatn!>2n1. We will show thatA(n) is true for alln1. For the base case, notice that 1! = 11 = 211:Thus,A(1) is true. Assume thatA(k) holds. Thus,k!>2k1. We will show thatA(k+ 1) is true by showing that (k+ 1)!<2k. Notice that (k+ 1)! = (k+ 1)k!(k+ 1)2k1: Sincek1, thenk+ 12. So, (k+ 1)2k122k1= 2k:Thus,A(k+ 1) is true. So,n!2n1for alln1. (d) Sincen!2n1, then we have that12 n11n!for alln1. Comparing term-by-term, we have that n X k=11k!nX k=112 k1: Since 10! = 1, we have that S n=nX k=01k!1 +nX k=112 k1: 2 (e) From Calculus II, we recognize nX k=112 k1=n1X j=012 j=n1X j=0 12 j as a geometric series. Thus, n1X j=0 12 j =112 n112 <1112 = 2: Thus, 1 +nX k=112 k1<1 + 2 = 3: (f) By (b),Snis a monotone sequence. By (d) and (e), we have thatSn<3 and is thus bounded. Thus, since every bounded monotone sequence converges,Snconverges. (g)

Question 3.In class, we learned that a sequence inRkis convergent if and only if it is Cauchy. We have

previously proven using the denition of convergence that the sequence x n=1n

converges (to 0). Thus, it should also be Cauchy. In this problem, we will prove directly that it is Cauchy.

(a) Letn;m2Z+. Show that1n 1m <1n +1m (b) Use (a) to show thatxn=1n is a Cauchy sequence. To do so, given an" >0, nd anNsuch that for alln;m > N,1n 1m <1n +1m

Solution 3.

(a) We will show this inequality by showing that 1n 1m <1n 1m <1n +1m

For the second inequality, notice that since

1m <1m , then 1n 1m <1n +1m

Similarly, it's clear that

1n <1n ;, so we get that 1n 1m <1n 1m Combining this gives the two inequalities, which is equivalent to 1n 1m <1n +1m 3 (b) Let" >0. By the Archimedean principle, there exists anNsuch thatN"2 >1. Thus,1N <"2 . Thus, for alln;m > N, we have that1n ;1m <1N <"2

Thus, by (a),

jxnxmj=1n 1m <1n +1m <"2 +"2

Thus,xn=1n

is a Cauchy sequence.Question 4.Consider the sequence of partial sums given by S n=nX k=11k 2: We will show thatSnconverges by showing it is Cauchy. (a) Ifn;m2Z+withm > n, show that jSmSnj=mX k=n+11k 2: (b) Show that 1k

2<1k(k1)fork2.

(c) Show that mX k=n+11k(k1)=1n 1m As a hint, think abouttelescoping seriesfrom Calculus II. (d) Use the above to show that jSmSnj<1m +1n (e) Use (d) in a proof to show thatSnis Cauchy and thus converges.

Solution 4.

(a) Since all the terms in the sum are positive andm > n, thenjSmSnj=SmSn. The terms inSmSn are those terms up tomexcluding the rstn. Thus, we have that jSmSnj=mX k=n+11k 2: (b) Notice thatk(k1) =k2k < k2. Since all terms are positive, we can cross-divide to get 1k

2<1k(k1):

4 (c) Notice that

1k(k1)=1k

1k1:

Thus, when adding up the terms in

m X k=n+11k(k1)=mX k=n+11k 1k1; all cancel except for the term 1n and1m . Thus, m X k=n+11k(k1)=1n 1m (d) Putting the above together, we have that jSmSnj=mX k=n+11k 20. By the Archimedean property, there exists anN2Z+such thatN"2 >1. Thus, 1N <"2 :Question 5.Letf;g:R!Rwitha;L;M;k2R. Furthermore, assume that lim x!af(x) =Land limx!ag(x) =M:

Give an"proof to show the following:

(a) lim x!akf(x) =kL (b) lim x!af(x) +g(x) =L+M

Solution 5.

(a) We will prove this in cases:k= 0 ork6= 0. In the rst case, ifk= 0, then we wish to prove thatkf(x) = 0, the zero function, has limit 0. So, given" >0, let= 1 (or any other positive number, really). Thus, for all 0Thus, lim x!akf(x) =kL: Next, assume thatk6= 0. Let" >0. Since limx!af(x) =L, there exists asuch that for allxsatisfying

0 jf(x)Lj<"jkj: Thus,jkj jf(x)Lj< "and thusjkf(x)kLj< ":So limx!akf(x) =kL. 5 (b) Let" >0. Since limx!af(x) =L, there exists af>0 such that for allxsatisfying 0Similarly, since lim

x!ag(x) =M, there exists ag>0 such that for allxsatisfying 00. Then, for allxsatisfying 0